G is the Cantor set, $X$ is a countable set of G hence GX is dense












0












$begingroup$



Exercise: Let $G$ be a Cantor set.



a) Prove that $G$ has a countable dense subset.



b)If $X$ is a countable subset of $G$ then $Gsetminus X$ is dense in $G$




a)
It is know that the Cantor set for example the ternary Cantor set has the same cardinality of $mathbb{R}$ despite its length is $0$. Since $mathbb{Q}$ are dense set in $mathbb{R}$ and $G$ is a subset of $mathbb{R}$ with uncountable elements hence $Gcapmathbb{Q}$ must be a countable dense subset of $G$.



b) I have no concrete idea on how to taclkle the question of countability and density in an abstract set $X$.



Questions:



1) Is my proof insofar right? If not. Why not?



2) How is b) solved?



Thanks in advance!










share|cite|improve this question









$endgroup$












  • $begingroup$
    For the first question: the cantor space $2^{omega}$ is clearly a Polish space, so you conclude. To see that it is Polish, consider that it is the countable product of the space ${0,1}$ with the discrete topology and this is a Polish space, so it is Polish.
    $endgroup$
    – Lei Feima
    Jan 25 at 20:52












  • $begingroup$
    for the second question: $X$ countable implies that $2^{omega}setminus X$ is uncountable.Then it is dense if and only if, for every open set $A$, $Acap (2^{omega}setminus X)neq emptyset$ , but then an argument from infinite combinatorics show you that it cannot be empty.
    $endgroup$
    – Lei Feima
    Jan 25 at 21:10












  • $begingroup$
    Not every Cantor set in the reals contains any rationals. The srtandard one contains plenty of them, and they do form a dense set. It does require some reasoning. Just compactness of $C$ implies it's separable.
    $endgroup$
    – Henno Brandsma
    Jan 25 at 22:34










  • $begingroup$
    Let $H$ be the set of all irrational numbers. Since $mathbb Q$ is a dense set in $mathbb R$ and $H$ is an uncountable subset of $mathbb R$, can you conclude that $Hcapmathbb Q$ is a countable dense subset of $G$?
    $endgroup$
    – bof
    Jan 26 at 0:55
















0












$begingroup$



Exercise: Let $G$ be a Cantor set.



a) Prove that $G$ has a countable dense subset.



b)If $X$ is a countable subset of $G$ then $Gsetminus X$ is dense in $G$




a)
It is know that the Cantor set for example the ternary Cantor set has the same cardinality of $mathbb{R}$ despite its length is $0$. Since $mathbb{Q}$ are dense set in $mathbb{R}$ and $G$ is a subset of $mathbb{R}$ with uncountable elements hence $Gcapmathbb{Q}$ must be a countable dense subset of $G$.



b) I have no concrete idea on how to taclkle the question of countability and density in an abstract set $X$.



Questions:



1) Is my proof insofar right? If not. Why not?



2) How is b) solved?



Thanks in advance!










share|cite|improve this question









$endgroup$












  • $begingroup$
    For the first question: the cantor space $2^{omega}$ is clearly a Polish space, so you conclude. To see that it is Polish, consider that it is the countable product of the space ${0,1}$ with the discrete topology and this is a Polish space, so it is Polish.
    $endgroup$
    – Lei Feima
    Jan 25 at 20:52












  • $begingroup$
    for the second question: $X$ countable implies that $2^{omega}setminus X$ is uncountable.Then it is dense if and only if, for every open set $A$, $Acap (2^{omega}setminus X)neq emptyset$ , but then an argument from infinite combinatorics show you that it cannot be empty.
    $endgroup$
    – Lei Feima
    Jan 25 at 21:10












  • $begingroup$
    Not every Cantor set in the reals contains any rationals. The srtandard one contains plenty of them, and they do form a dense set. It does require some reasoning. Just compactness of $C$ implies it's separable.
    $endgroup$
    – Henno Brandsma
    Jan 25 at 22:34










  • $begingroup$
    Let $H$ be the set of all irrational numbers. Since $mathbb Q$ is a dense set in $mathbb R$ and $H$ is an uncountable subset of $mathbb R$, can you conclude that $Hcapmathbb Q$ is a countable dense subset of $G$?
    $endgroup$
    – bof
    Jan 26 at 0:55














0












0








0





$begingroup$



Exercise: Let $G$ be a Cantor set.



a) Prove that $G$ has a countable dense subset.



b)If $X$ is a countable subset of $G$ then $Gsetminus X$ is dense in $G$




a)
It is know that the Cantor set for example the ternary Cantor set has the same cardinality of $mathbb{R}$ despite its length is $0$. Since $mathbb{Q}$ are dense set in $mathbb{R}$ and $G$ is a subset of $mathbb{R}$ with uncountable elements hence $Gcapmathbb{Q}$ must be a countable dense subset of $G$.



b) I have no concrete idea on how to taclkle the question of countability and density in an abstract set $X$.



Questions:



1) Is my proof insofar right? If not. Why not?



2) How is b) solved?



Thanks in advance!










share|cite|improve this question









$endgroup$





Exercise: Let $G$ be a Cantor set.



a) Prove that $G$ has a countable dense subset.



b)If $X$ is a countable subset of $G$ then $Gsetminus X$ is dense in $G$




a)
It is know that the Cantor set for example the ternary Cantor set has the same cardinality of $mathbb{R}$ despite its length is $0$. Since $mathbb{Q}$ are dense set in $mathbb{R}$ and $G$ is a subset of $mathbb{R}$ with uncountable elements hence $Gcapmathbb{Q}$ must be a countable dense subset of $G$.



b) I have no concrete idea on how to taclkle the question of countability and density in an abstract set $X$.



Questions:



1) Is my proof insofar right? If not. Why not?



2) How is b) solved?



Thanks in advance!







general-topology proof-verification proof-writing cantor-set






share|cite|improve this question













share|cite|improve this question











share|cite|improve this question




share|cite|improve this question










asked Jan 25 at 20:12









Pedro GomesPedro Gomes

1,9132721




1,9132721












  • $begingroup$
    For the first question: the cantor space $2^{omega}$ is clearly a Polish space, so you conclude. To see that it is Polish, consider that it is the countable product of the space ${0,1}$ with the discrete topology and this is a Polish space, so it is Polish.
    $endgroup$
    – Lei Feima
    Jan 25 at 20:52












  • $begingroup$
    for the second question: $X$ countable implies that $2^{omega}setminus X$ is uncountable.Then it is dense if and only if, for every open set $A$, $Acap (2^{omega}setminus X)neq emptyset$ , but then an argument from infinite combinatorics show you that it cannot be empty.
    $endgroup$
    – Lei Feima
    Jan 25 at 21:10












  • $begingroup$
    Not every Cantor set in the reals contains any rationals. The srtandard one contains plenty of them, and they do form a dense set. It does require some reasoning. Just compactness of $C$ implies it's separable.
    $endgroup$
    – Henno Brandsma
    Jan 25 at 22:34










  • $begingroup$
    Let $H$ be the set of all irrational numbers. Since $mathbb Q$ is a dense set in $mathbb R$ and $H$ is an uncountable subset of $mathbb R$, can you conclude that $Hcapmathbb Q$ is a countable dense subset of $G$?
    $endgroup$
    – bof
    Jan 26 at 0:55


















  • $begingroup$
    For the first question: the cantor space $2^{omega}$ is clearly a Polish space, so you conclude. To see that it is Polish, consider that it is the countable product of the space ${0,1}$ with the discrete topology and this is a Polish space, so it is Polish.
    $endgroup$
    – Lei Feima
    Jan 25 at 20:52












  • $begingroup$
    for the second question: $X$ countable implies that $2^{omega}setminus X$ is uncountable.Then it is dense if and only if, for every open set $A$, $Acap (2^{omega}setminus X)neq emptyset$ , but then an argument from infinite combinatorics show you that it cannot be empty.
    $endgroup$
    – Lei Feima
    Jan 25 at 21:10












  • $begingroup$
    Not every Cantor set in the reals contains any rationals. The srtandard one contains plenty of them, and they do form a dense set. It does require some reasoning. Just compactness of $C$ implies it's separable.
    $endgroup$
    – Henno Brandsma
    Jan 25 at 22:34










  • $begingroup$
    Let $H$ be the set of all irrational numbers. Since $mathbb Q$ is a dense set in $mathbb R$ and $H$ is an uncountable subset of $mathbb R$, can you conclude that $Hcapmathbb Q$ is a countable dense subset of $G$?
    $endgroup$
    – bof
    Jan 26 at 0:55
















$begingroup$
For the first question: the cantor space $2^{omega}$ is clearly a Polish space, so you conclude. To see that it is Polish, consider that it is the countable product of the space ${0,1}$ with the discrete topology and this is a Polish space, so it is Polish.
$endgroup$
– Lei Feima
Jan 25 at 20:52






$begingroup$
For the first question: the cantor space $2^{omega}$ is clearly a Polish space, so you conclude. To see that it is Polish, consider that it is the countable product of the space ${0,1}$ with the discrete topology and this is a Polish space, so it is Polish.
$endgroup$
– Lei Feima
Jan 25 at 20:52














$begingroup$
for the second question: $X$ countable implies that $2^{omega}setminus X$ is uncountable.Then it is dense if and only if, for every open set $A$, $Acap (2^{omega}setminus X)neq emptyset$ , but then an argument from infinite combinatorics show you that it cannot be empty.
$endgroup$
– Lei Feima
Jan 25 at 21:10






$begingroup$
for the second question: $X$ countable implies that $2^{omega}setminus X$ is uncountable.Then it is dense if and only if, for every open set $A$, $Acap (2^{omega}setminus X)neq emptyset$ , but then an argument from infinite combinatorics show you that it cannot be empty.
$endgroup$
– Lei Feima
Jan 25 at 21:10














$begingroup$
Not every Cantor set in the reals contains any rationals. The srtandard one contains plenty of them, and they do form a dense set. It does require some reasoning. Just compactness of $C$ implies it's separable.
$endgroup$
– Henno Brandsma
Jan 25 at 22:34




$begingroup$
Not every Cantor set in the reals contains any rationals. The srtandard one contains plenty of them, and they do form a dense set. It does require some reasoning. Just compactness of $C$ implies it's separable.
$endgroup$
– Henno Brandsma
Jan 25 at 22:34












$begingroup$
Let $H$ be the set of all irrational numbers. Since $mathbb Q$ is a dense set in $mathbb R$ and $H$ is an uncountable subset of $mathbb R$, can you conclude that $Hcapmathbb Q$ is a countable dense subset of $G$?
$endgroup$
– bof
Jan 26 at 0:55




$begingroup$
Let $H$ be the set of all irrational numbers. Since $mathbb Q$ is a dense set in $mathbb R$ and $H$ is an uncountable subset of $mathbb R$, can you conclude that $Hcapmathbb Q$ is a countable dense subset of $G$?
$endgroup$
– bof
Jan 26 at 0:55










1 Answer
1






active

oldest

votes


















1












$begingroup$

A compact metric space is separable, so the Cantor set is too. For a concrete dense set in the Cantor set: take all the end points of the intervals in the construction: at stage $0$ we have $[0,1]$ and we add $0$ and $1$ to the dense set; at stage $1$ we have $[0,frac13]$ and $[frac23,1]$ and we add $frac13,frac23$ to the dense set; at the next stage we add $frac{1}{9}, frac29, frac79, frac89$ etc. In the end we get all "rationals" in the ternary expansion: all those numbers (that have only $0$ and $2$'s in a ternary expansion) that are eventually periodic.



All non-empty open subsets of $C$ are uncountable (they all contain sets homeomorphic to the whole Cantor set, even), so all of them intersect the complement of any countable set.






share|cite|improve this answer











$endgroup$













    Your Answer





    StackExchange.ifUsing("editor", function () {
    return StackExchange.using("mathjaxEditing", function () {
    StackExchange.MarkdownEditor.creationCallbacks.add(function (editor, postfix) {
    StackExchange.mathjaxEditing.prepareWmdForMathJax(editor, postfix, [["$", "$"], ["\\(","\\)"]]);
    });
    });
    }, "mathjax-editing");

    StackExchange.ready(function() {
    var channelOptions = {
    tags: "".split(" "),
    id: "69"
    };
    initTagRenderer("".split(" "), "".split(" "), channelOptions);

    StackExchange.using("externalEditor", function() {
    // Have to fire editor after snippets, if snippets enabled
    if (StackExchange.settings.snippets.snippetsEnabled) {
    StackExchange.using("snippets", function() {
    createEditor();
    });
    }
    else {
    createEditor();
    }
    });

    function createEditor() {
    StackExchange.prepareEditor({
    heartbeatType: 'answer',
    autoActivateHeartbeat: false,
    convertImagesToLinks: true,
    noModals: true,
    showLowRepImageUploadWarning: true,
    reputationToPostImages: 10,
    bindNavPrevention: true,
    postfix: "",
    imageUploader: {
    brandingHtml: "Powered by u003ca class="icon-imgur-white" href="https://imgur.com/"u003eu003c/au003e",
    contentPolicyHtml: "User contributions licensed under u003ca href="https://creativecommons.org/licenses/by-sa/3.0/"u003ecc by-sa 3.0 with attribution requiredu003c/au003e u003ca href="https://stackoverflow.com/legal/content-policy"u003e(content policy)u003c/au003e",
    allowUrls: true
    },
    noCode: true, onDemand: true,
    discardSelector: ".discard-answer"
    ,immediatelyShowMarkdownHelp:true
    });


    }
    });














    draft saved

    draft discarded


















    StackExchange.ready(
    function () {
    StackExchange.openid.initPostLogin('.new-post-login', 'https%3a%2f%2fmath.stackexchange.com%2fquestions%2f3087573%2fg-is-the-cantor-set-x-is-a-countable-set-of-g-hence-g-x-is-dense%23new-answer', 'question_page');
    }
    );

    Post as a guest















    Required, but never shown

























    1 Answer
    1






    active

    oldest

    votes








    1 Answer
    1






    active

    oldest

    votes









    active

    oldest

    votes






    active

    oldest

    votes









    1












    $begingroup$

    A compact metric space is separable, so the Cantor set is too. For a concrete dense set in the Cantor set: take all the end points of the intervals in the construction: at stage $0$ we have $[0,1]$ and we add $0$ and $1$ to the dense set; at stage $1$ we have $[0,frac13]$ and $[frac23,1]$ and we add $frac13,frac23$ to the dense set; at the next stage we add $frac{1}{9}, frac29, frac79, frac89$ etc. In the end we get all "rationals" in the ternary expansion: all those numbers (that have only $0$ and $2$'s in a ternary expansion) that are eventually periodic.



    All non-empty open subsets of $C$ are uncountable (they all contain sets homeomorphic to the whole Cantor set, even), so all of them intersect the complement of any countable set.






    share|cite|improve this answer











    $endgroup$


















      1












      $begingroup$

      A compact metric space is separable, so the Cantor set is too. For a concrete dense set in the Cantor set: take all the end points of the intervals in the construction: at stage $0$ we have $[0,1]$ and we add $0$ and $1$ to the dense set; at stage $1$ we have $[0,frac13]$ and $[frac23,1]$ and we add $frac13,frac23$ to the dense set; at the next stage we add $frac{1}{9}, frac29, frac79, frac89$ etc. In the end we get all "rationals" in the ternary expansion: all those numbers (that have only $0$ and $2$'s in a ternary expansion) that are eventually periodic.



      All non-empty open subsets of $C$ are uncountable (they all contain sets homeomorphic to the whole Cantor set, even), so all of them intersect the complement of any countable set.






      share|cite|improve this answer











      $endgroup$
















        1












        1








        1





        $begingroup$

        A compact metric space is separable, so the Cantor set is too. For a concrete dense set in the Cantor set: take all the end points of the intervals in the construction: at stage $0$ we have $[0,1]$ and we add $0$ and $1$ to the dense set; at stage $1$ we have $[0,frac13]$ and $[frac23,1]$ and we add $frac13,frac23$ to the dense set; at the next stage we add $frac{1}{9}, frac29, frac79, frac89$ etc. In the end we get all "rationals" in the ternary expansion: all those numbers (that have only $0$ and $2$'s in a ternary expansion) that are eventually periodic.



        All non-empty open subsets of $C$ are uncountable (they all contain sets homeomorphic to the whole Cantor set, even), so all of them intersect the complement of any countable set.






        share|cite|improve this answer











        $endgroup$



        A compact metric space is separable, so the Cantor set is too. For a concrete dense set in the Cantor set: take all the end points of the intervals in the construction: at stage $0$ we have $[0,1]$ and we add $0$ and $1$ to the dense set; at stage $1$ we have $[0,frac13]$ and $[frac23,1]$ and we add $frac13,frac23$ to the dense set; at the next stage we add $frac{1}{9}, frac29, frac79, frac89$ etc. In the end we get all "rationals" in the ternary expansion: all those numbers (that have only $0$ and $2$'s in a ternary expansion) that are eventually periodic.



        All non-empty open subsets of $C$ are uncountable (they all contain sets homeomorphic to the whole Cantor set, even), so all of them intersect the complement of any countable set.







        share|cite|improve this answer














        share|cite|improve this answer



        share|cite|improve this answer








        edited Jan 25 at 22:32

























        answered Jan 25 at 21:06









        Henno BrandsmaHenno Brandsma

        112k348121




        112k348121






























            draft saved

            draft discarded




















































            Thanks for contributing an answer to Mathematics Stack Exchange!


            • Please be sure to answer the question. Provide details and share your research!

            But avoid



            • Asking for help, clarification, or responding to other answers.

            • Making statements based on opinion; back them up with references or personal experience.


            Use MathJax to format equations. MathJax reference.


            To learn more, see our tips on writing great answers.




            draft saved


            draft discarded














            StackExchange.ready(
            function () {
            StackExchange.openid.initPostLogin('.new-post-login', 'https%3a%2f%2fmath.stackexchange.com%2fquestions%2f3087573%2fg-is-the-cantor-set-x-is-a-countable-set-of-g-hence-g-x-is-dense%23new-answer', 'question_page');
            }
            );

            Post as a guest















            Required, but never shown





















































            Required, but never shown














            Required, but never shown












            Required, but never shown







            Required, but never shown

































            Required, but never shown














            Required, but never shown












            Required, but never shown







            Required, but never shown







            Popular posts from this blog

            Mario Kart Wii

            What does “Dominus providebit” mean?

            Antonio Litta Visconti Arese